You are on page 1of 33

IP

WP
HP* :
a

^ 1
%

pprORCED CONCRETE DESIGN 229


1e
XJ
BEA M REIN FOR CED As * p bd
FOR TEN SION
( tension reinforcement index )

a
fc p bd fy ’

,
o V
Mu = 0(0.85) 0 85 fc -
F U
**
- Mu = 0 fc' co bd
[
.
AS fy
d - 0.85 fc' b( 2 )
r p
P bd fy
[d - 2 fc- (Q.85) b
]
r p b2d 2 fy T
Mu = 0 fc co [ 2
bd
' 2 -
(0.85) b J
•O -O-O- - co bd
r 2 l
Mu = 0 fc' co bd -
[
2 (0.85) J
0 85/ ’
AAu = 0 fc ' co bd 2 ( l - YJ )
Mu = o fc' © bd 2 ( 1 - 0.59 co)
C=0.85/ tai

JW,
Rn = © ( 1 - 0.59 co)
fc'

Rn = coefficient of resistance
C = 0.85 fc' ab Mu = 0 bd 2 Rn
(compressive force of concrete )
T = As fy Rn '

0 bd 2

( tensile force of steel reinforcem ;nt )

Hi
IFh = C [®5
T=C
(steel ratio )
As fy = 0.85 fc' ab
Asfy
Mu * o As fy ( d - a / 2)
0.85 fc' b
( ultimate moment capacity)
Mu = 0 C (d - a / 2 )
Mu = 0 0.85 fc' ab ( d - a / 2) Mn = Asfy (d - a / 2 )
..
Mu " —
o 0.85 fc' As fy b
SSfc -V
- ( d a / 2) '
( nominal moment capacity )
SI
230

and steel
^
REINFOR
c d
A. Design for dimension 0.003 £, + 0.003 U*3
reinforcements. c6 0 003
n si

Given data : e + 0.003


Ultimate moment M, CO
0.003 '
Concrete strength fe' "
-L + 0.003
Yield strength of steel . . . . f

c„
E .
0,003(200,000) n,
Required : ”
a) Width of beam b 7 f + 0.003(200,000) Sek
b) Effective depth of beam . . . d 600 d A,
c) Steel reinforcement needed At c6 =
* fi + 600
N=
C=T Is
0.85 f ’ ab = f,
* 0 003 o 85fc -
0- 85 f.’ R cb b = Af
yil 1
c
p6 X
A
bd
..
Ste
ft
ft
0.85 f/ Bctb = p( bdf>
/
» •
i

b
. 0.85 f ’act
)
P

» f, d
_ 0.85 r 8 (600) d
8. h)v
Stepl . Compute balanced steel ratio : Pb "
In a balanced section, steel starts yielding * <
, f, + 600) fcr
when concrete just reaches its ultimate strain _ 0.85 f/ R 600 (Ur
-
capacity and commences to crush. At the Pt f ( fy + 600) (Pi
start of failure , the permissible extreme fiber
compressive strain is 0.003, while the tensile (balanced steel ratio)
strain in the steel equals the yield strain 0 85 Vs 600
Pt -~ f (
Wir
f
, 600 + fy ) &
E . Assume p - 0.5 pb
pt Co
(0 = —r l
fie
* OSSfc -

*'
1
« Step 2. Assume min. thickness of be
S«hply supported ~z
BT-
,
. A ’ ix - T- ,
One end continuous
16
L
fia
** /
So<h end continuous
21
Cantilever beams
* eiNWBCI
U»o steel covering
h * d + 62.5
d * h 62.5-
62.5

Compute for "b"u8ing :


f ' bdJ
m

-
(o (1 0.59(u)
rONCRETE DESIGN

A
0.003 0.85fc’
231

-
(d a/ 2 )

Mu 0
*,
£t T= Sf,
*
Step 3. Steel reinforcement needed
a) Tension Reinforcement index
A, = pbd
Select no. of bars: p=
F -bd
A (
= — 2
d N
N = no. of bars
d = diameter of bars
b) Depth of compression block
Step 4. Check for pmt and p
^ Compare with p
P = 075 P ^
> * p < pmax steel bars will yield
Pim Pntn p > psteel bars will not yield
1.4 *
P > P^ = T 0 =1
0.85 fc' ab = A fy

a M
B. Investigation of Beams Reinforced 0.85 fc ' ab
for Tension if Steel will yield:
(Under Reinforced ) c) Ultimate moment
(pmln < p < pmax) a = lic
C
_a
Given data : IJ
Width of beam b e, _ 0.003 ,
» * 0.7 + <e -0.002)25<V3
Effective depth of beam . . d d-c c
Steel reinforcement At When:
Concrete strength f' £, > 0.005 X
Yield strength of steel . . . . f Use 0 = 0.90 I Other 0 a 0.65 (£ -0.002)250/3

Span of beam L
When: carolled coo®

^
f l Required : £, > 0.002
a) Tension reinforcement index , . w ,
£ < 0.005

^ Depth of compr ession block . a


c) Ultimate moment
Mu
Use 0 = 0.65 + ( £ - 0.002) ,
f) Safe concentrated load
®
Coefficient of resistance
P
R„ When:
,
£ < 0.002
Use 0 = 0.65
232 REINFORCED n
s) Balanced Steel ratio
Nominal Moment Capacity
Mn = A, fy (d - 3 /2 ) L d M Ir C s a i* m
Ultimate moment capacity:
=
Ma At 0 (d - a/ 2)
B = 0.85 - -
005 ft' 30)
TrJ t

d) Coefficient of resistance
r
but not less than 0.65

Pb
_ 0.85 f ' B 600 /lr
:
I
Mu = 0 <u bd2
(1 - 0.5 9<a)
U 600 ,
R
"
~
M
0 bd
2
*')

b) Depth of compression block


A,
e) Safe concentrated load

M
8 4
' S
p < pmt steel bars will not yield

Use: f
_ 600 (d - c) a) Depth
c
C. Investigation of Beams Reinforced
t for Tension if Steel will not yield: C=T
(Over Reinforced ) 0.85 f ' ab = A f a = Gc (d
(pmln ^ Pmtx < p )
b) Balant
c C =T
Given data : Solve fore: (quadratic equation) O Sf ' ab
Width of beam b a = Bc ^
Effective depth of beam . . . d 0.8
Steel reinforcement
Concrete strength
As
f'
Yield strength of steel . . . . f
c) Ultimate moment
Mu = 0 A f (d - a/2) ks t

n Span of beam L .
D Balanced Condition
Required :
^^ '
mai

a) Balanced Steel ratio .


Given data :
Width of beam b
E*
b) Depth of compression block . .
c) Ultimate moment
a Effective depth of beam . . . d P* ®
y Steel reinforcement
Concrete strength
A,
f' C
1 5,
SHi
ZX f " 1 | ° C= 85 /C'«4
Yield strength of steel . . . . f
Span of beam L ^17

Dr Required : V

*
a) Depth of compression block 3
e
* T=AsFy b) Balanced steel area
c) Ultimate moment
for balanced condition
233

m b 0.85 fc ' Tension Steel Yields


&
30) r
3. Problem:
(d -al 2 )
5 A rectangular concrete beam has a width of
r 250 mm and a total depth of 450 mm. It is
2
reinforced with a total steel area of 187 5 mm
placed at an effective depth of 375 mm.
fc' = 27.6 MPa, fy = 414.7 MPa.
c
ssionbiOCk i ® Determine the depth of compression
block.
® Determine the moment capacity reduction
fr /E , 0.003 factor.
will not yield (D Determine the safe live load that the beam
could carry in addition to a dead load of

3 a) Depth of compression block :

,
80Od
f + 600
20 kN/m if it has a span of 6 m.

Solution:
M

a = lie (depth of compression block ) © Depth of compression block :

»)(d - c) b) Balanced steel area


c C= T
equation) 0.85 T ab = As6 X
OSSVab
fy
,
A =1873
/r -
> ' < «•
As
^ '
As# = balanced steel area
Assuming the steel yields:
T=C
c) Ultimate Moment for Balanced
,
A fy = 0.85 fc’ ab V
Condition:
1875(414.7) = 0.85(27 -6)(a) 250)
a = 132.58 mm
< %
Max. A - 0.75 Asb
J!t»nate moment for balanced condition: @ Moment capacity reduction factor.
.. b
..d
*
M = 0 A fy (d - a/2) a = UC
132.58 = 0.85 C
A,•
Check: C = 155.98
.. . 0.8 5 f ' li (600)
V p
n
> "
T(MOH ) (baiancedsteelrali0)
0.001
. .
L
1= ^
bd
C =155.98!

t/
- -»5
:i 9 o:
75

VI

Av

M.
234 REINFORCED CONCRETE

E
91Q 09
, 0.003
IRSQft
E { = 0.0042125 > 0.002 but < 0.005
Mu
_ WUL
8
2
^ r
2
Wu (6)
e, =1 200.2 r
E.
414.7
8
W„= 44.48 kN/m Pm
C , = 200 000 Pm
£y = 0.0020735 < 0.0042125 = 1.4 DL + 1.7 LL p-
Wu
44.48 = 1.4(20) + 1.7 LL
Steel yields: st»
LL = 9.69 kN / m
0 = ,
0.7 + (£ -0.002 )250/3
0.90
c

Spiral
Tension Steel
c 0.70 Does Not Yield
i 0.65 /
0 = 0.65 + -
(£ 0.002) 250/3
«
i Other
4. Problem : f :

' 1
A rectangular beam has a width of 280 mm f :
Compression Transition Tension and an effective depth of 500 mm. It has a
controlled controlled
,
e == 0.005 reinforcement area of 4072 mm2 at the bottom.
0.002
cV== 0.60 C=
SI =
c/d 0.375 Balanced steel ratio p t> = 0.026. fc' = 25 MPa,
fy = 414 MPa.
0.8!
,
Since e is between 0.002 and 0.005, 0.8!
tiiis value is within the transition range © Determine the depth of compression
between compression controlled section block. 505
and tension controlled section. © Determine the tensile force carried by the c 2

steel bars. c=
Use 0 = 0.65 + (£ - 0.002)

0 = 0.65 + (0.0042125 - 0.002)

0 = 0.834
,
250
3
^ ® Determine the resisting moment capacity
of the beam, checked for moment capaoty
reduction factor.

i
a=
a=
as

Solution:
© Depth of compression block.
© Live load it could support: .
0 S5fc

( ")
Mu = 0 M y d
if . S

Mu = 0.834(1875)(414.7)(375 - d=500
,v
I f

Mu = 200.2 x 106 N.mm 500

Mu = 200.2 kN .m *
s
I
T
X
T,
r.

DESIGN 235

P - bd
'
h1072 :0.029
® Resisting moment capacity of the
beam, checked for moment capacity
reduction factor.
p = (500)
280 Mu = 0 As fy (d - a/2)
= 7 $ Pb
L
Pimx
=
^ '

0.75 (0.026) = 0.0195


Mu = 0 T (d - a/2)
£, _ 0.003
PnB "
>
P Pmx (over reinforced) 193.92 306.08
£, = 0.0019 < 0.002
Steel does not yield: Use 0 = 0.65
= 0 T (d - a/2)
0.003 . £,
260 171
c d- c Mu = 0.65 (1547930) (500 - )
n Steel 0.003(d - c) 2
= 372,2 kN.m
ot Yield
f, = £sE
c
.
_ 0.003(d - c)(200,000) Balanced Condition of
M

f
c Beams Reinforced for
as a width of 2S3 f,
_ 600 (d - c) Tension
) of 500 mm. IthL
. c
1072 mm2 atthebcq|- C=T 5 . Problem :
, = 0.026. fe' - Z 0.85 f ‘ab = A f A beam has a width of 300 mm and an
4072(6i00
jepth of f
con?®
0.85(25)(0.85)c(280) =
2
-
5057.50 c = 122160000 - 2443200c
c ^ effective depth of 500 mm. fc' = 28 MPa .,
fy = 414 MPa. Es = 200,000 MPa .

2
c + 483.08c - 241542.3 = 0 © Determine the depth of compression block
sile force carried Pi for a balanced condition.

sssd
c = 306.08 mm
a = IJc ® Determine the balanced steel area
required.
a = 0.85(306.08)
® Determine the moment capacity for
a = 260.17 mm (depth of compression block)
maximum steel area requirements for a
balanced condition.
® Tensile force carried by the steel bars.
Solution :
ssion bloC ^ f . 600 (d - c) © Depth of compression block for a
o# c balanced condition.
0.003 . 600 (500 - 306.08)
f
a:
306 08 300 0.003 0.85 fc‘
f f = 380.14 MPa < 414 MPa
;
295.86

-'
iM ‘
Tensile force: 500
T = As fs
?. v. 14
T T = 4072(380.14)
T = 1547930 N
Aab Ml
Et f /
U 15 7.93 kN ^ ab y

*
Si*

mr

REINFORCED CONCRETE DES ,


236

c
600 d 5 - A Problem:
^ 0.85
f 900 A rectangular beam having a width of 30
600(600) and an» w
dliu 01 . .
effectivwe depth of -45
WM ryy
0
0 mm. ^ 0.85
C~
414 + 600
reinforced with 4- 36 mm in diameter ^
fc’ = 30 MPa fy = 270 MPa . 3 si
C » 295.86 Es = 200000 MPa.

© Compute the balanced steel ratio of fe


aa (ic beam. Mn =
a * 0.85(295.86) © Compute the nominal moment capacity i
the beam.
a - 251.48 mm Mn =
® If the value of fc' is reduced by 50X,
compute the percentage of the reduced
nominal moment capacity of the beam. Mn =
UI Balanced steel area required.
'

Mn =
C=T
0.85 fc ab = Asb fy Solution:
p Perce
0.85(28)(300)(251.48) = Asb (414) © Balanced steel ratio. morm
Asi ~ 4337 mm2
0.85 fc' R (600)
pb " fy (600 + fy)
( 3) Moment capacity for maximum steel 0.85(30 )(0.85 )(600 )
area requirements for a balanced pb ~
270(600 + 270) T
condition. p=
= 0.0544 5
A = 0.75 Asti Pb
P = 0j
(max . steel area for balanced condition)
p = 0.
A, = 0.75(4337) © Nominal moment capacity.

2
At = 3253 mm
0- 85/c ’

£ = £, = 0.002 < 0.005


Use 0 = 0.65 Pb * S;
Mu = 0 A, f (d - a/2)
!
0,
7
Mu = 0. 65( 32 53)( 41 4 ) (500 - — ~ )
Mu = 327.62 kN.m
C=T
-ORCED CONCRETE DESIGN 236-A

0.85 fc' ab = As fy
Pmax " 0.75 Pb
J* 1
<
J (36) 2(4)(270)
* fl«
>lP 0.85(30)(a)( 300) = pmax = 0.75(0.0277)

a 143.70 Pmax = 0.0208

v. As
( |)
= Asfy d - P = bd
Anient
is reduced J Mn = f ( 143
(36) 2(4)(270) 450 - 2
.70
p=
f (36)2(4)
rrtage of 300(450)
oadty of ihetsy j Mn = 415.7 x 106 N.mm steel does not
p = 0.03016 > 0.0208
Mn = 415.7 kN.m. yield

@ Percentage of the reduced nominal


moment capacity of the beam.
Since the steel will not yield, locate the
neutral axis from the top of the beam.
:i
M
As
P=M
bd 0.003
4!
m
0) P=
f (36)2(4) c

' 300(450)

p = 0.03016 < 0.75(0.0554)

p = 0.04155 (under reinforced)


Z 450 - c
£s =/y /Es
= 270/ 200000
= 0.00135
larity -

' 0.003 0.00135


0.85/r 0.85 fc' (J 600 c
= 450 - c

#1
Pb fy(600 + fy)
1.35 - 0.003c = 0.00135c
0.85(15)(0.85)(600 )
270(600 + 270) 0.00435c = 1.35

^ Pb = 0.0277 c = 310.34
a = lie
r
p
m

CONCRETE’
236-B
REINFORCED

MPa
C=T
Note: 6 = 0.85 for fc' 5 30
0.85 fc ' ab = A, fy
3 = 0.85(310.34)
a = 263.79 mm. K
0.85(20.7)0(400) = - (28f (4 (4 , I
( |)
Mn = Asfy d - a = 145.09 mm
^
|Zi )
Mn = J 2
(36) (4)(270)(450 - ^ a = Bc
145.09 = 0.85 c
Mn = 349.7 x 106 N.mm
c = 170.69 mm
Mn = 349.7 kN.m
0.003
Percentage of the reduced nominal
moment capacity: 170.69
349 7
Percentage = x 100
Percentage = 84.1%
429.31
&
£ .s
5-6 Problem :
A reinforced concrete beam has a width of 0.003
400 mm and an effective depth of 600 mm. It
429.31 170.69
is reinforced for tension with 4 - 28 mm 0
ft bars. fc’ = 20.7 MPa, fy = 414.6 MPa. £, = 0.00754
»
* © Determine the percent increase in nominal
m

t
moment if the depth is increased to
700 mm.
© Determine the percent increase in nominal
e"
’ i414.6
*ft*
moment if fc’ is increased to 27.6 MPa.
© Determine the percent increase in nominal
£, 200,000
moment if the steel is change to £y = 0.00207
4 - 32 mmo.
£s > £y (steel yields)
Solution:
© Percent increase in nominal moment if the Nominal moment if d = 600 mm
depth is increased to 700 mm.
0003 0.85fc’
.
M = « A, ft d - ( |) J
c

600
C
2
M„ = 0.90( 7i:/4)(28) (4K I
41 |
4.6 H
d -a!2
M„ = 484755959 N.mm
- 4-28 mm*
, M„ =484756 kN.m
Es T =A sfy
400
/ >
%
. .

EggCONCRETE DESIGN 236-C

Honing m om en t if d = 70 0 m m ® Percent increase in nominal moment if the


- steel is change to 4 - 32 mm 0.
Mr * eA, fy (d a/2)

% M = 0. 90 ( 7rM)( 28) (42


)(41 4. 6) 70 0(- C=T
0.85 fc ' ab = A, fy
= 57 66 60 66 3 N.m m
Ma J
= 576661 kN.m
0.85(20.7)(a)(400) = —4 (32) (4X414.6)

a = 189.51 mm

Percent inc re as e in no m in al m om en t :
(576661 - 484756)
% inC re3Se =
484756
10 0 Mn = 0 AS fy (d - |)
( )(32) (4X414.6) (600 -
^
% increase = f 8.96%

^
2
M„= 0.90

M„= 606490 kN.m

@ Percent ina ea se in no m in al m om en t if fc’


is increased to 27.6 MPa. % inaease in nominal moment:
'

(606490 - 484756)
C=T % increase = — <00
0.85 fc * ab = As fy % inaease = 25 f %.
2
0.85(27.6)(a)(400) = (28) (4)(414.6)
J
a = 108.82 mm

.
M = ® A, C d ( f)
= 0.90(I)(28)’(4)(414.6) (600 - ®? )
Mn = 501423048.1 N.mm
M„= 501423 kN.m

Percentage inaease in nominal moment:


-
li? : W cf -, (50 14 23 - 48 47 56)
484756
% increase = 3.44%
236- D

5-C Problem
REINFORCED CONCRETE DES
I
^^ :

® Amount of steel needed. :


A rectangular beam has a width of 250 mm ic
and an effective depth of 575 mm. The beam C=T
is reinforced for tension to cause the strain in 0.85 fc’ ab = As fy
the tension steel to be 0.005 just as the
maximum strain in the concrete reaches 0.85{20.7)(183.28)(250) = As (414.6)
0.003. As = 1945 mm 2

fc = 20.7 MPa «40


,
f = 414.6 MPa

© Determine the depth of compression 3 Ultimate strength of the beam -


block.
® Determine the amount of steel needed.
® Determine the ultimate strength of the
beam.
Mu ( |)
= 0 Asfy d- ASvJ "
C= T
Mu = 0.90(1945)(414.6) (575 - 0.85 f/
Solution:
i , . © Depth of compression block Mu = 350.8 kN.m B - z
0.003 0.85fc’
-
A, = 14

r
1.85(27
: a = 104
J 4, ., *
-
1 5-D Problem
d -af 2
A rectangular beam has a width of 250
He
As and a total depth of 640 mm. The beam
reinforced with 3 - 25 mm 0 placed with Bp 122
T =A fy
steel covering of 65 mm from the bottom of tl^ B*
b =250 mm

By ratio and proportion


beam. fc’ = 27.6 MPa. Assume A 615 gra E
60 steel with fy = 414.7 MPa. * ^ ^
0.003
~
_
0.005 © Determine the distance from
c 575 - c compression fiber to the neutral axis of
1.725 = 0.008 c beam.
c = 215.63 mm © Determine the nominal moment of
beam.
ucam
^
u
a=8c ® Determine the percentage increase jo jjl l°03
3 = 0.85 (215.63) nominal strength of the beam if| J

a = 183.28 mm increase the steel reinforcement


100%
{REINFORCED CONCRETE DESIGN 236- E

Solution :
® Nominal moment of the beam.
ext rem e
.
Q Dis tan ce from com pre ssio n fiber
to the neutral axis of the beam. (
M = A, l, d - 5)

.
0.003 0.85 fc '
M = 1472.6(414.6) 575 (
r
M u = 319.4 kN.m
S 6»C -
d an

3- 25 mm 9
offr« beani. ® Percentage increase in the nominal
Es T= sfy strength of the beam if we increase the
250 mm
* steel reinforcement by 100%,

f) ? Assume steel will yield: 0.003 0 851V


1

C=T C

K414.6) (575 -H 0.85 fc' ab = As f,


C

575
J d 41 / 2
71 ; A = j (25) (3)
4
2
As = 1472.6 mm
0.85(27.6)(a)(250) = 1472.6(414.7) E.S r =As / y
a = 104.12 mm 250 — A

C=T
& a = fic
0.85 fc‘ab = A, fy
1 of Si 104.12 = 0.85 c
0.85(27.6)(a)(250) = 2945.2(414.7)
c = 12150 mm (distance from extreme
a = 208.25 mm
compression fiber to neutral axis)
tB
( -

Checkifsteel wiflyield: 0.003 M u = A s f y (d - 1)


122.50
M„= 2945.2(414.6) 575 (
dlSt

nof
3nC
!e
^ - £s / . T „
,
Mu = 575 .11 kN.m

l^ so '
es % increase = 80%
^| ^
^ < c = 0 -0 11 1 >e = = 0.0021
«
of V HI
*
290,000
^ «*
-
S< lyields
*

mm

REINFORCED CO NC RE TE DESI 0
236-F

M A, (d - f )
^ i
T
jit ' '

5#

P
cre te bea m has a
A rectangular reinforced con 0.9
width of 400 mm with an effe ctiv e dep th of Mn = 246 3(414 .6) (600 - 145.09)
600 mm. It is reinforced wit h 4 - 28 mm 0
M„= 538.62 kN.m
* 46*6
fc’ = 20. 7 MP a, ; C
bars at the bottom ,

fy = 414.6 MPa.
P-
© Determine the per cen tag e inc rea se in Mu = 466
2
nom ina l mo me nt if the ste el rein for cem ent ,
When A = - (32) (4) Ch ec k for
is changed to 4 - 32 mm a . m
® Determine the percentage increase in A, = 3217 mm
2 Mu = ; —
nominal moment if the effective depth is | 40(!
increased to 700 mm. HI
8
® Determine the percentage increase in Mu = 405
nominal moment if fc is increased to C=T
M 27.6 MPa. 0.85 fc' ab = A, fy Irate rr
ftebeai
0.85 (20.7) a (400) = 3217(414.6)

Solution: a = 189.51 mm
9 Cut-of
© Percentage increase in nominal moment if suppo
i the steel reinforcement is changed to
4 - 32 mm a.
». 0.003 0.85fc ’
2
9r i

)
w

*
V
'
c

- :
/ a
Mn = 3217(414.6)

M = 673.88 kN.m
(eoo -
^ * , 23

3-28 mm
. T -Asfy
Percentage increase in M.
- 40 o
/0 increase = <
^^ »
53862 °°
\
2
= ~ (28) (4) % inaease = 25.11%
&
A, = 2463 mm 2
a . 38.!
2)

C=T
0.85 fc ' ab = A, fy \
0;
0.85(20.7){aK400) = 2463(414.6)
a = 145.09 mm
*
i
H8B1

PIETE DESIGN 237


a
2
( )Ultim ate mom ent capacity of the beam:
2 Wu »40 kN/m
Mu = 0l(d - ) 1 Z
6) 600.

•' .m
< 145.09)
Mu = 0.90 Asfy (d - ~ )

( ) 2
Mu = 0.90 j ( 25) (4)(414.7) 675 - ( — H)
*~ X

180
^ n 180

Mu = 466.4 x 10 N.mm
®
Mu = 180(x) -
(32)J (4) Mu = 466.4 kN.m
Check for actual moment: 240.3 = 180x - 20x 2
2 WuL 2 x - 9x + 12.02 = 0
2

Mu = 9 ± 5.74
8 X=
40(9) 2 2
Mu : x = 1.63 m. (cut off point of 2 bars)
8
Mu = 405 kN.m < 466.4 kN.m (safe) Alternate Solution:

»' =40 kN/m
fr Ultim
lIHii ate moment capacity of
the beam is Mu = 466.4 kN.m
I00) = 3217(414.6) | 1 L |
X
2 -25 mm 0 .j J
-
Cut off point of two of the bars from the
9m
support: 180 180
•i
450
0.85 fc ' 0.003
- ) -c
iv „=40 kN/m 2-25 mm 0
' 189
2 O; p
c =38.57 /
)
6) (eoo"T
. ^ 2 -25 mm 0
dJl2/f T
~

/ rf-c=655.72 L
x
2- 25 mm a

T =Asfy Ej 9m
l.m
Remaining steel bars is 2 - 25 mm a
: As = ( 2 164.7
ease inM» j 25) (2)
.
),
53862 00 ^ = 981.75 mm2 </ a
573

,11%
^
538-62 C= T
0-85 fc’ ab = As fy
085
240.3 .

°-85(27.6)(a)(450) = 981.75(414.7)
a = 38.57
4.5 4.5 '
Using squared property of parabola:

Mu : 0 As fy (d - a/2)
405 _ 1647
2 " 2
(4.5) a
Mu = 0.90(981 75)(
(
414 7) 675 - a = 2.87 m. from center
= 240.3 x 106 N.mm x = 4.5 - 2.87
Mu = 240.3 kN .m x = 1.63 m. (from support)
'
*
IfJkuf . .

REINFORCED CONCRETE DEs|g


238

Determination Whether 1963.5


P - 300 (380)
Compression Bars or
Tension Bars is needed p - 0.017
pma - 0.75 pb = 0.023
Q|Q3s SZZESE12u3l
^
A 12 m. simply supported beam is provided by
an additional support at mid-span. The beam
p < pmax
Only tension bars is needed.

has a width of b = 300 mm and a total depth Therefore the beam needs only
h = 450 mm. It is reinforced with 4 - 25 mm 0
at the tension side and 2 - 25 mm 0 at the
bars as specified in the problem.
C=T
^
compression side with 70 mm cover to
centroid of reinforcements. Fc’ = 30 MPa, 0.85 fc' ab = As fy
fy = 415 MPa. Use 0.75 pt, = 0.023. 0.85 (30)(a)(300) = (^/4) (25)2 (4X415)
a = 106.52 mm
© Determine the depth of the rectangular (depth of rectangular stress block)
stress block.
© Determine the nominal bending moment,
Mn.
® Nominal bending moment:
® Determine the total factored uniform load
including the beam weight considering
moment capacity reduction of 0.90.
Mn = Asfy d - (
S
?

^
Mn = - (25)J (4)(415) 380 -
Mn = 266.2 x 106 N.mm
( ^
y
Mn : 266.2 kN.m
Solution:
© Depth of the rectangular stress
block:
® Total factored uniform load includinj
0003
beam weight:
0.85fe '

</=380
310 -
c
A
unmniHiiuuH
S
|s
R
J
50 e
* T

I. ' k
Check if compression bars is
needed.
As
8=1 . *
P a TT
bd

. ^
384 El
A> =

A, =
f1963.5
(25)2 (4
8
"
48
L
El
mm2 fv.
Po
0 | ft
REINFORC E D C O N C R ET E DE SI GN 239

5 wL " PLa
4

© Evaluate the curvature % due to nominal


3M ® 48 El flexural strength Mn of the cross section in
0 023 SwL
P= radians per m.
8
® Determine the magnitude of the cross
2R + P = wl
sections curvature ductility ratio
' ' '****
S 5 , 5 wL
2R = wL • — g—
bsam 3 wL_ 300
r
OD
2Rl
l £c
rf n the proi

ify
'
0) = (R/4) (25)i
^ R = — wL
3

M, = R (| - W |
) ( ) ( t )
450
AL
315.10
.

w 450-x

I 2 2
- -
.
45=962.5 mm2
o
3 wL wL er=? I
^ angular stress 16(2)
2
8
2
3 wL - 4 wL
MB 32 Solution:
ing moment: wL 2
© Curvature y/y due to moment My which
MB = - produces initial yielding of tension steel:
~ ) Mu = 0.90 Mn Es
2 = 0.9 0 ( 266 .2 ) n"
'

Mu Ec
(4)(415) 380 ( Mu = 239 .58 kN .m
200000

.m
0* N mm

239.58 =
^
32
.
w(12) 2
4700 >/ fc'
200000
239.58 4700 V242
32
w = 53.24 kN 1 m n = 8.65 say 9
j uniform - 102

M
| = n/1s (450 - x)
CURVATURE of ^
BEAMS 150x2 = 9(962.5)(450 - x)
2
150x + 8662.5X - 3898125 = 0
2
x - 57.75x - 25987.5 = 0
A rectangular reinforced concrete beam has a x = 134.90 mm
r width of 300 mm and an effective dep th of 450
450 - x = 315.10 mm
i mm It is reinforced for. tension at the bottom
with a total steel area of 962.5 mm'v fc = 24.2
MPa fy = 345 6 MPa . Es = 200000 MPa .

® Evaluate the curvature vfy due


,0 £y "
345.6
200000
moment My which pro duc es init ial yie ldin g ey = 0.001728
f the tension steel in radians per m.
°
40
REINFORCED
c
BEAM REINFORCED
e
Curvature y/y = 45Q - x
. FOR COMPRESSION
o.oo

0.001728 A. Design for dimension and steel


Vy ~ 315.10
reinforcements.
y/y = 0.00000548 radians/mm
Given data :
\ffy = 0.00548 rad /m.
Design moment Mu P' \
Width of beam b
Effective depth of beam . . . d pbd
© Curvature y/u due to nominal flexural Concrete strength f'
strength Mn: Yield strength of steel . . . . f
Required :
0- 8S/C a) Moment to be carried by P- 7

-
£ =0-003
2 *
c compression bars M2
b) Total steel area in tension . . As
450 Assun
c) Steel area in compression . . As '

mm
1
I* T=C
«JSmrai
As =9

T=Asf , d'
0.85fc '
C , Cj=4s7s '
A =/
C=T

.85 f
°
1

R Asfy = 0.85 fc'ab (d -d )


id«n\
962.5(345.6) = 0.85(24.2)(a)(300)
a = 53.90 & I© a=

a = Rc Tr* sif, TrzAsi!>
=0
a) Moment carried by the compressionk
,=M
53.90 = 0.85c
To obtain a reduction factor of 0.90, the
c = 63.42 mm max. reinforcement ratio corresponding
b)
W/ = —
0.003 to a net tensile strain of 0.005 is equal to: Tots
a=G
0.003 P = 0.85 ft V / 0.003
0 003 j
Vu 63.42 f ' 0.003 + 0.005' Ca '
if /u = 0.0000473 rad/mm From the strain diagram: Frorr
y/u = 0.0473 rad/m died
0.003
% y
3 Curvature ductility ratio = — C
Wy
: d
CO -
* fMk
CDR - 8.63
Et =0.005 0.003
1*
BSpRCEDCONCRETE DESIGN 241

C d
;i
"Sfon
0.003 0.003 + 0.005
£= 0.003
d 0.003 + 0.005
1) When e >
E .
Compression bars will yield

As fy = 0.85 fc a b
'
(V = f ) ,
A# fy = 0.85 fc' R c b
1
beam .
H
V H=
p
^ bd
2) When e;<
E .
Compression bars will not yield
d
pbdfy = 0.85 f ’li c b 600 (c - d ) 1

Use : r
;f 0.85 T lie c
teel . . f P
i
M2 = 0 As2 fy (d - d ) 1

riedby
0.85 fc ’ B (0.003) .A M2
2
P ,
82 =
III 0 fy (d - d )
1

rs fy (0.003 + 0.005)

A, = A + Aa
itension . . A Assumed : p = 0.85 liM — —+ — )
fy ' 0.003 0.005'
ipression . . A

0.85fc ’
As1 = pbd c) Steel area in compression :
C=T If compression bars will yield
Cf
Ci + '
* AS2 = As'
\ V
0.85 f ' ab = A , fy AS2 = a ;
_L_
( d -aP

&
-\ ^
a= -

, ,
M =0A
0.85 f ' b
fy (d - a/2) .^
If compression bars will not yield
As f vr
,
r =^i/ A
M2 = MU - M, where : f ' =
600 (c - d')
c
ii
bj Tota/ sfee/ area in tension :
A '=
’ ^V
a = lic
a
c= -

& '

g'
li
From the strain diagram,
check whether compression bars will yield
_
0.003
,
B. Compression Bars Will Yield
Given data :
Total steel area in tension . . At
Steel area in compression . . At’
tgram c - d'
e
*
._ c
0.003 (c - d )
c
1
Width of beam : b
Effective depth of beam . . . d
Concrete strength f'
Yield strength of steel . . . . f
Span of beam L

i
40.
005 ^
'
'
A

REINFORCED CO
242
c) Safe uniform liveload
Required : }
W, l
a) Depth of compression block a M„
b) Ultima!’moment capacity M
c) Safe uniform liveload
wu = 1.4D + 1.7L

it axiki carry t-t


C. Compression Bars Will Not Yield
t L 0.85 fr ‘
rf ’ T
**
—r r , £gJ>
C Given data :
-

V rrfT Total steel area in tension . . Af
{ d -d )’ Steel area in compression . . At '
(rf /2
'
i ^
© © Width of beam
Effective depth of beam . . . d
b

T ,=ASIf } Ti=Asif , Concrete strength f'


a) Depth of compression block : Yield strength of steel . . . . f
Check if compression bars is really needed. Span of beam L
Required:
K
bd a) Depth of compression block . . a
P , - 0.75 p , b) Ultimate moment capacity . . . Mu
If P > P (compression.bars are needed)
c ) Safe uniform liveload
( it could carry LL
repression bars will yield if the
blowing condition occurs: b '
0.85fc '
0.85 fc ' Rd' 600
P ^
fy d (600 - fy )
%
F (i 4 \
bd (<d -a / 21
f,' = fy (steel in compression yields)
T = C + C2,
- ©i 8 ft i
r.T=V>
As fy = 0.85 fc ’ ab + As ’ fy
Solving for a: a) Depth of compression block :
Check if compression bars is really needed pOfe
0.85 fc ' b
p
-
bd
h,
Pnax = 0.75 pb
b) Ultimate moment capacity :
a = Bc ,fP >P (compression bars are needed)
M„= C1 (d - | -
) + CJ (d d'j Compression bars will not yield if the
following condition occurs:
,
C = 0.85 fc ' a b
n .r > * M5 fc' Rd' 600
.
C2 = As’ fy
fy d (6d0 - g
Nominal moment capacity:

Mn = 0.85 fc' ab (d |
- ) + A,' fy (d - d') fs fy (steel in compression does not
Ultimate moment capacity: *
UseV =Hii I ®N
Mu = oMn ^
NCRETE DESIGN 243

,
,
T = C C?
-
A, f = 0.85 f ’ a b + A,' f,'
{

*1
b

a
0.85 fc -
Cl
C2~AS fy

Solving fa a: V
h A, f A;V - d
(d-a / 2 ) (d-d )

0.85 fc ' b
& &
I
b) Ultimate moment capacity :
-4 i-- TrASlfy TflAS2f ,
: a = Sc
- a) Depth of compression block :
P
(d -|) * C2 (d - f) (
Balanced Steel Ratio,
C, = 0.85 fc’ a b (1) If compression bar: will yield:
C2 = A,' V - 0.85 fc' R (600)
Nominal moment capacity: Pb P
fy (600 + fy )
Vj Mn = 0.85 fe' ab (d - |) + A,' f/ (d - < T)
(2) If compression bars will not yield:
fy Ultimate moment capacity: — ~_ 0.85 fc' R (600) +t , V
Mu = 0 Mn Pb P
(600 + fy ) fy
. . . LL
c) Safe uniform iiveload : c=
^
600 d
600 + fy
fc W„L ?
0.05 (fc' - 30)
8 B = 0.85 -
7
Wu = 1.40 + 1.7L (but not less than 0.65)
8 = 0.85 for fc’ < 30 MPa
l -all ) a = IJc
D. Balanced Condition for Beams
Reinforced for Compression and
*'
SI > Tension: b) Balanced Steel Area :
Given data : ,
C + C2 = T
s wm Total steel area in tension . . A, 0.85 fc ' ab + As ' fy = Asfy
Steel area in compression . . A,' (A, - A,') fy = 0.85 fc' ab •
Width of beam b
AS - A ' = AS1
5

Effective depth of beam . . . d


Conaete strength f'
Yield strength of steel . . . • fy
Span of bean L
- _ 0.85 fc' ab-
Atf = ' AS
r
Ast = Asi + AS2
Required : c) Maximum Amount of steel area permitted:
a) Depth of compression block . a
Max. As = 0.75 ft bd + As'
, b) Balanced Steel area
_ 0.85 fc' R (600)
v " '
c) Maximum Amount of steel
^ fy (600 + fy )

area permitted:
REINFORCED CONCRETE DESl
243-A
Solution:
© Ratio of depth of compression block t ,J
distance of the neutral axis from the tar ' 0 s

supported beam has a width of the beam.


A simply
mm . It b c }

300 mm and an effective depth of 330 0.003


c 1 '

is reinforced at the bottom with a steel


reinforcement area of 2464 mm 2. Assuming

steel covering to the centroid of the


reinforcement is 70 mm. fc' = 34 MPa,
d
\
f7.
N A.
T . 11

fy = 415 MPa. At . L i Value

© Determine the ratio of the depth of - = ft


compression block to the distance of the c
neutral axis from the top of the beam. . 0.85
© Determine the balanced steel ratio. ^
0.05(34 - 30)
© Determine the depth of compression 8 = 0.85 -
block.
7 a=B<
II = 0.82
© Determine the total compressive force 117.94
carried by concrete.
© Balanced steel ratio:
| c = 14
® Determine the value of the moment
reduction factor 0. Pb “
_
0.85 fe‘II 600
186.1]
fy (600 + fy ) le, = o
© Determine the design strength of the
beam.
Pb
_ 0.85(34)(0.82)(600)
® Determine the distance of the neutral axis 415(600 + 415)
B = 0.(
from the top of the beam if it is reinforced Pb = 0.034
with a steel area pf 1232 mm2, at the top
* in addition to the existing steel area of 0 = 0.1

TOP 2464 mm2 at the bottom.


© Determine the stress of the compression
bar.
® Depth of compression block: |8 = 0 ,

300 0.003 %0
© Determine the design strength of the 0.85/c’ esigr
beam using moment reduction factor 0 =
0.90.
3
= *B
143.83 / k, «

</=330 /1
® Determine the concentrated live load it
, * * \ h l 0
could carry at the mid-span in addition to a yt =>246 ran? 1 JJ ,
dead load 'of 20 kN/m
including its own 70
'

T* A , f, £s

weight for a span of 6 m.

C=T
0.85 fc' a b = As fy
0.85(34)(a)(300) = 2464(415)
a = 117.94 (depth of compres ^^i
f:.
* 1NF0RC ED CO NC RE TE DE SIG N 243-B

’SI |* com pres sive forc e carr ied by conc rete : ® Distance of neut ral
beam if additional
axis
stee l
from
bars
the
are
top
plac
of
ed
the
at
C s 0.85 tc’ a b
the top:
J%3 | C = 0.85(34 X117.94)(300)
i C = 1022540 N
300 0.003
i C * 1022.54 kN
:
0.85/f ’

LfTS-rJli-: PLA
ip +
C2 c

If Value of moment reduction factor 0:


330

,
A *2464 nun?
300-C
0.003 70 T , Tl

C=143.83

M
-
330 C=186.17

a = Bc
117.94 = 0.82 c
c = 143.83
Check whether compression bars will yield:

Compression bars will not yield.


I
*
£, _
'
0.003
when p p < - , 0.85 fc' B d' 600
186.17 143.83 ,
f d (600 - fy )
i!
£ , = 0.0039 < 0.005

(250) Compression bars will yield.


,-
0 = O.65 + ( £ 0.002)
3
, 0.85 fc’Bd' 600

^

0 = 0.65 + (0.0039 - 0.002)


L

-m.
3

0 = 0.81 TL

A
P= b d
.
* M. = BA f, (d -
Design strength of the beam:

My
til
. |) 2464
P = 300(330)

,AM K = 0.81(2464)(415) (330 - p = 0.025

= 224.5 x 10* N.mm


- 224.5 kN.m

5'
% m

REINFORCED CONCRETE DESIG IJ


243-C

too 0 85/r '


,
^
P =
bd
1232
300(330) =
rf 330
If
__ .
,

a

NA
__ ;
c,
>d -an.)
p' = 0.0124
A , 246* nun'
»

70 r
p - p* = 0.025 - 0.0124
p - p' = 0.0126 V:
-
0.003(c 70)(200,000)
C

0.85 fc’ (3 d' 600 _ 0.85(34)(0.82)(70)(600) V


600 (c - 70)
G
f d (600 - f ) 415(330)(600 - 415)
C, + C2 T, + T2
=
; 0.85 f/ l3 d' 600
H = 0.0393 C1 + C 2 = T
: fyd (6O0 - g
0.85 fe' a b + A,' f,' = As fy
0.0126 < 0.0393
0.85( 34)(a)(300) + 1232 f,' = 2464(415)
*
8 ’
8670a + 1232 f,' = 1022560
I Steel in compression does not yield:
a = (3 c
if
V* fy a = 0.82 c
r 0.003
HC :3 = 10
c
70 E, ' 8670(0.82) c +
1232( 60
°c
C -70
IH- c - 39.86c - 7278.25 = 0
2

I* / c = 107.54

0.003 _ g; ® Stress of compression bar


c c 70 - 600 (c - 70)
0.003 (c - 70) V
£ c
‘ 3
600 (107.54 - 70)
5
107.54
Vs 209.45 MPa < 415 MPa
RCED CON 243- D

De sig n str en gth us ing re du cti on fa cto r


® MU = 0 M„
,
c ' 0 = 0.90
M„= 0.90(285.68)

.- —
NA 300 0.85/c’
Ml 70
U Mu = 25 7.1 1 kNjn

J

a C2
A ,' 2464 ram

r
71
' rf =330
.
-
(d an )
-
(d d' )

( 200,000) =2464 mro?


70 ^ Tl T2
® Live load it could carry.

‘a = Bc
3 = 0.82 (107.54)

a = 88.18
1
nmiinmiiiHi
wDL =20 kNVm
H

: =\ \ ,
C = 0.85 fc ' a b
6m

+ 1232 f,' = 2

= 1022560
«d C , = 0.85(34)(88.18)(300)
,
C = 764521 N U
'
& w m L v .4).
M
_ P(6)(1.7) , 20(36)(1.4)

C2 = A;V
C2 = 1232(209.45)
Mu = 51.42 kN
232( 6000 C2 = 258042
i

8.25 '
° Mn = C, (d - j) + C 2 (d - d )
1

M„= 764521 330 - M( )


3r :
SSi nb + 258042(330 - 70)
}
° s
M„= 285.68 x 10 N.mm
M„= 285.68 kN.m
P

244 REINFORCED C
Stir I Areas Are Required
P^ ~
_
Assume value of p :
0.85(0.85)(27.6)(0.003)
0, Problem : 414.7(0.008) ' A

A rectangular beam that must carry a service -


p 0.018
live load ot 36 kN / m and a calculated dead
load ol 16.4 kN /m on a 5,4 m , simple span is
limited In cross section for architectural
A,, pbd -
A „ = 0.018(250)(400)
reasons to 250 mm and 50 mm total depth.
Assume that the tension steel centroid will be „
A = 1800 mm 2
Co^ pressw
100 mm above the bottom face of the beam V= ‘ ' E
and that compression steel if required will be
placed 82,5 mm below the beams top surface,
C=T
0.85 fc' a b = A„ fy
-
!=
fs
f = 0.00175
• 350 MP
& f,/ * 27,6 MPa , „
f « 414.7 MPa .
0.85(27.6)(a)(250) = 1800(414.7)
<D Determine the remaining moment to be ,
Mj = Atf
0 f
carried by the compression steel. a = 127.27 mm x 106 :
® Determine the steel area In compression . 37.48
A = 306 mi
m i -fo
3 Determine the steel area in tension
( )
*
Solution:
M, = „fy (d - )
0A
1 2
A, , = As' f
CD Remaining moment to be carried by the M , = 0.90(1800)(414.7) (4OO - ~ 306(414.7) =
compression steel .
M , = 226 kN.m \' = 363 mn
B ® Steel area ir
:: Wg ' 1.4 DL + 1.7 Li
W„ " 30(1, 7) + 1.4(15.4) =
M? MU - M , As = Asi + As
l W„ * 72.56 kN/m Mj = 264.48 - 226 As = 1800 + :
M 2 = 38.48 kN .m I As = 2108 mi
250
,
a, • B C

wm
400 © Steel area in compression .
<5,
dan <& dd
a = Rc
0.003
r/ " Vr 127.27 0.85 c = tegular be
=
c 149.73 149.7:
WUL
8
?

C, _
0,003
87
N * ?
nsio
" at
M . '
250.27 149.73
C, = 0.005
250.27

M „ * 264.48 kN. m , i mPression


>

To obtain a reduction factor of 0,90, the max.


e; _"
0.003
e Aii


reinforcement ratio corresponding 87.23 149.73
tf
to a net tensile strain of 0.005 is equal to: e, ’ = 0.00175 < ey = ~
“ [L J £y
414.7
200 000
I
th,

et = 0.00207 Pan
'
"
! Value of p . BH&SRCEP CONCRETE DESIGN 245

, 47fo35 ipS ; 250


i 84 -• *
* •

0 S5 fc

c?
S'. Solution:
Depth of compression block .
1
400 j -a n * \M , dd 300
bd
<5.
0 003

1A
rW 5!]
°18< 250)(400) — # — Tt - ^Sjfy T2 = AS fj
2
-
l 2t m

300 mm*' 600 .


Compression steel does not yield 6 - fnrfi
*
V = £s’E, III e

ab =M v
= 0.00175 (200 000)
,f ’= 350 MPa 2
6)(a)( 250) = 1800(4147:

A, = 7 (32) (6)
4
.27 mm M, = oAs, f (d tf) , - A, = 4825 mm
2

37.48 x 10* = 0.90 A , (414.7){ 400 - 62.5)


A = 306 mm
1 *
^
T;= C , P bd

el MSVV 4825
90( 1800)(414.7) 400 - ( 306(414.7) = AS' (350)
:
P
300(600)
A, =
' 363 mm p = 0.0268
26 kN.m
@ Steel area in tension P„- = 0.75 p „
As = As1 + AS2 0.85 V 6 600 '
Pt ‘
64.48 - 226
As = 1800 + 306 f, (600 + f ) ,
As = 2 f 06 mm2 0.05 (V - 30)
8.48 kN.m 6 = 0.85 -
7
8 = 0.817
Compression Bars will yield 300 V
0.003 0.85fc

rea in com

= 0.85 c
pre ss

,
' "a
0 '

^ width of 300 mm
|A rectangular beam has a
„ and an effective depth of 600 mm. The steel
62

600
•— hv Y /
- — f i x'

:-i - 4
±\
V7a

>r*t
,
(d

©
-uafl
/2 ) W

*
mm 2 and
1.73 49 area of tension at the bottom is 4762 e
that of the compression side at the top is *
987.5 mm2. Steel covering is 62.50 mm from
0.003 the compression face of the beam. P>
0.85(34 6)(0.817)(600)
= 414.7(600 + 414.7)
14073 A = 0.034, fc' = 34.6 MPa, = 414.6 MPa
005 ^ pt = 0.03426
© Determine the depth of compression P = 0.75(0.03426)
block. = 0 025695
_ 0.003 ® Determine the design strength using a
P
=
14973 reduction factor of 0.90. Compression bars is needed:
< " £, ® Determine the concentrated live load at
1.00175 the mid- span in addition to a dead load of P >P
0.0268 > 0.025695
20 kN/m including its own weight if it has a
4147 _ span of 6 m.
!00 000
. 00207
7/ > REINFORCED CONCRETE D£
s

Mu = oMu
Mu = 0.90(1031)
= 927.9 kN.m

(D
Concentrated live load at the mid-span
in addition to a dead load of 20 kN
/m
including its own weight if it has a span
of 6 m.

®
Pes' 9n stre 9th using a
factor of 0.90." reduction
0.003 -
-
n o«„

d-d lilt
bo

s#
BBJfCED CONCRETE DESIGN
I ab /
2
^(
d- .
+ c? (d
2J ^ ^ Solution:
247

Depth of compre ssion block


q ^
WJi
I $ .
ab
W 6)(
'
'
18064K3(yf
* ^ As = - (28) (4)

2
. sv/
. __F'
87 N

5(414.7)
As - 2463 mm
As' = -- (28f (2)
4
2

z r

I2 As' = 1231 5 mm* T = Ci Ca


A fr = 085 If ab r As V
m (6oo - 1® , A.. *2463(415) = 0>85( 36)(a 300)
+ 407132 (& ^
302- 62i;
o-
K

~-
—fcd-
2463
P 330(330
12315
me - ^
70)
c

_ 738900(c - 70)
x 106 N.mm ;
p = 0.0248 1022145 = 7650a +
c
I />«, - 075 p, 73890 c:
1031)
.p = 0.75(0031)
pw = 0.023
. 1022145 = 7650(0.85c)
157.19c = cr^ + 113.63(c - 70)
c ^^ *
IkN.m
pmax
-
c* 4356c - 7954.10 = 0 1

43.56 ± 183.61
ited live load atttei c=- = 113.59
Therefore reinforce merit for comoression 2
l to a dead load of II ‘ a=8c
is needed .
its own weight if i la 3 = 085 (11359)
Compression bars wii not / ed if -
a = 96.55 (deptfi of compression block)
3;
L 1.7 PL .
p-p <
055 Id 8 <7600
+ f/ d (600 - fy)
! 4 % Design strength using 0.90 as reduction
0.85 (30)(0.85)(70/ yX3) factor:
4(20)(6f’ , (

“~ 4 ^ 415 (330) (600 - 415) 4


8 p - p’< 0.036 300
-
OtH i
rkN "
Tl •rq
A(
p= C/
'd
«qr ~
r h ay/ '
.

_
i i' „
, @ ^
7/
©
jorted beam 7?

t the bottom
# w ^ p - p> ( 0.036
0.0248 - C.C124 < 0.036
0.0124 < 0,036
*
C. = 055 f ' ab
le beam. C, = 055(30X9655X300)
nforcement C, = 738 60750 N
f the beam w>I. Therefore compression bars wfl not yield.
.

OOmmaod Vl * ^
fy
415 MP3 .
|
*
0003
o.m C2 = A,' V

- S^ ht ^
= = £ Z f,* = £,' Ft
*
Jk C c - 70

1
. W03(c 70) .
J c c TO
-
' * 0# 5'
£
L ....

A .. . 600 (113.59 - 76;


113.59
rthedes / OoM V=0 f ' = 23C 25 « 41 3 -

rr- -
m
'

REINFORCED CONCRETElia
248
^l
^
|Np
(

c, Solution:
C, = 1231.5(230.25) + C.
© Depth of compression block fw f Ci
a
,
C = 283553 balanced condition. 0.85 fc
0.85 -
(20
- )
,( | 250
M, = C d 0.003 0.85 fc* +

,
M = 738607.5 330 -
M, = 208 x 10 N.mm
(
^ )
62.5 |

</=625 mm. _ _
c
m ASb = 47 :

JE
®
625-c
M 2 = C2 (d d ) - 1
Maximurr
M2 = 283553 (330 - 70) es permitted
:
Mj = 73.7 x 10 N.mm ' Max As
.
,
MU = 0 (M + M2 ) 0.8£
’ E, ‘
Mu = 0.90(208 + 73.7)
= 253.53 kN.m £> =
400
200 000
* T
ey = 0.002
® Concentrated live load at mid-span: 0.003 0.002 = 0.0Z
PL WL 2
c 625 - c
Mu (1 -7) ( I '4) . 0.005 c = 0.003(625) Max. As =
4 8
c = 375 mm Max . As :
11 (1.4)
253.53 =
^
P = 50.01 kN
4

Balanced Condition of
(1.7) +
^ 8
Check:
c=
600 d

Beams Reinforced for


c
_ fy + 600
600(625) |rectangular
A
I fd has an eff
Compression 400 + 600
| MPa,
fc = 25
c = 375 mm
12. Problem: a = Rc
P
a = 0.85 c
A reinforced concrete beam has a width of
a = 0.85(375) »
E 2 *
0*mm
*i
250 mm and an effective depth of 625 mm. it
is reinforced for compression having a steel a = 318.75 mm f
M ° °
factor.
2

area As’ = 1250 mm2 with a steel covering of


62.5 mm measured from the center of the
steel reinforcement to the top most fibers of
© Area of balanced steel Asb for the
cross section.
css mine

.
the beam, fc = 20.68 MPa, fy = 400 MPa
E = 200 000 MPa.
Determine the depth of compression block
62.5 l
250
0.003 0-8lfI

312.5
375
'

ix " ne
for a balanced condition. *
< =625 mm
SSi(
!

© Determine the area of balanced steel Asb


for the given cross section. u
© Determine the maximum area of flexural
steel in tension permitted in the given
£,' _ 0.003
"

cross section as required by the NSCP 312.5 375


Specifications.
e;= 0.0025 > £ y = 0.002
( steel in compression yields)
*
MffiPORCEP CONCRETE DESIGN 248-A

%' * fy = 400 MPa Solution:


Ci + Cz = T ' 1
Moment capacity if At * 6000 mm2.
0.85 fe‘ ab + A,' fy = A8b fy T «C
1,85(20 ,68)( 318.75)(250 ) A, 1f = 0,5 (fab
+ 1250(400) = Asb (400) 6000(300) * Q.85(25)(a)(375)
ASb = 4752 mm 2 a * 225,88 mm
a * IJc
225,88 = 0,85 c
Maximum amount of tensile c = 265.74 mm
permitted .
steel
e, _
0,003
Max. A, = 0.75 (\ bd + A,' 184.26 265.74
et = 0.00208
.0.85 fc' ft (600)
fy (600 + fy ) VA
_
0.85(20.68)(0.85)(600)
’ 300
400 (600 + 400) ty " ,
200 000
I ft = 0.0224 Ey = 0.0015 < 0,00208 (steel yields)
Max . As = 0.75(0.0224)(250)(625) + 1250
Max . As = 3875 mm2
.H
f
Mu = 0 A, fy (d - a/2)

= ,90(6000)(300) (450 -
Mu 0

-
12 A Problem: Mu = 546 kN.m

A rectangular beam has a width of 375 mm Q) Depth of compression block if As' = 3000
and has an effective depth of 450 mm. mm2 is at the top of the beam.
i V * 25 MPa, fy = 300 MPa. _2Z*_ 0.85/, '
4[ |B <>V 1s'
"

if 0 Determine the moment capacity if it is


f reinforced with steel area in tension As = 450
•Iif " *

(d -a /2 ) (d -d ’ )
6000 mm2. Use 0.9 as moment reduction
factor. .
A ,*6aoo © &
I ® Determine the depth of stress block if steel Tl‘ASI / , Tl‘ st f,
*
in'
mm 2 is added
f compression As = 3000
'
. . jiff ,
with steel covering of 60 mm below the top 6000
If of the beam. y 0.036
i ® Determine the moment capacity of the bd 375(450)
a '% beam reinforced for tension and A' 3000
1» compression ,
d= -
P
bd
—* =
375(450)
= 0.0178
A 0.85 fr * a d* 600
575
am 0.003 when 1p - Hp’ >
fyy d (600 - fy )
0.85(25)(0.85)(60)(600)
II
265.74
-
0.036 0.0178 >
300(450)(600 - 300)
id am 0.0178 > 0,0161
I »4 26
A,+*600fi
r,
ip

248- B REINFORCED CONCRETE D

Compression bars will yield;


Solution :
Depth of compression block
f ' =f
A 3000
K , * V, = = ®
'
Atf = 3000 mm 2 ^
p
M 280 S °,
0.007
bd 280(510)
* «000 = A„+ 3000
a “ Af = 3000 mm .
* 2
0,85 ft' S (T 600
1

WVT, '(steel in compression will not yield)


0.85 T ab + A,' fy = A fy 0.05 (f/ 30) -
0.85(25)a(375) + 3000(300) = 6000(300) B = 0.85 -
7
a = 112.94 mm 0.05 (35 - 30)
6 = 0.85 -
® Moment capacity of the beam reinforced 7
for tension and compression . 6 = 0.81
Ml = Ast fy (d - a/2) 0021 - 0007 <
0 85 35)(0 8
- < g
Mi = 3000(300)( 450 - 112.94 /2) 400( 510X600 - 400)
Mi = 354.18 kN.m 0.014 < 0.0177
M2 = AS2 fy (d d ) — V f,
M2 = 3000(300)(450 - 60) *
e; _ 0.003
M2 = 351 kN.m
Mu = 0 (Mi + M2)
Mu = 0.90(354.18 + 351)
C
^
. O.OOstc - SO)
Mu = 634.66 kN.m es = c

12-B Problem:
V = e ' E, .
,, = 0.003(c - 50X200 000)
ft
A rectangular beam has a width of 280 mm ,, = —
600 (c - 50)
and an effective depth of 510 mm. Steel area '• . :
c

in tension As = 3000 mm2 and steel in /
compression As’ = 1000 mm2. Steel covering T = C + C2 ,
for steel in compression is 50 mm . fc’ = 35 A, fy = 0.85 fc' ab + Al' fs'
MPa, fy = 400 MPa. 3000(400) = 0.85(35)(0.81) c (280)
(D Determine the depth of comp
ression
block. x 1000(600)(c 50) -
® Determine the actual stress of c
compression bar. 1200000c = 6747.3c + 600000c -
3

@ Determine the design moment capacity of 6747.3c - 600000c - 30,000,000 = 0


2

the beam. Use 0.90 as moment reduction


factor.
2
-
c 88.92c - 4446.22 = 0
c= 88 .92 ± 16 0.2 9
,
I
Mg
- 0,85fc.' 0003
a—, 1 so! 2

V
»
A *
’ 1000
cr c = 124.60 mm
a=6c
510
p
© 0 -41 /2)1 \d 3n
000.
A 3 = 0.81(124.60)
a = 100.93 mm (depth of compresS*
£in
-X -

glNFORCnD CONCRETE DESIGN 249

® Ac tua l stre ss of com pre ssi on ba r .


a) Effective Width of Flange:
,, 600(c - 50)
m f = '
* c
600(124.60 - 50)
.
A FORT-SECTIONS:
V 124.60 Limits of the effective flange with
!0 fs' = 35 9.2 3 MP a "b" as controlled by NSCP:
) 1@ Design moment capacity of the beam. Us the
e sm alle st va lue of "b"

M2 = A;fs' (d - d')
M2 = 1000(259.23)(510 - 50)
'
Slab Flange
b
l/\ Uv-
M2 = 165.25 kN.m
u 5 ML
Web or
M, = C, (d - a/2) stem — d
M, = 0.85 fc ' ab (d - a/2)
'
i
11X .
M = 0.85(35)(100.93)(280)(510 - 100.93/2) b'
600 - 41] M, = 386.35 kN.m Spacing o( Beams |

Mu = 0 (M , + M 2
) 1
= 0.9 0( 16 5.2 5 + 38 6.3 5) © b= of span
Mu ^
. b = 16t + b„
M u = 496 .44 kN m ©
© b = center to center spacing
of beams
1'
i
ANALYSIS OF .
B FOR L- SHAPED FLANGE:
T-BEAMS b

A. Steel Area required with Neutral


Axis on the Flange: d
Given data :
C |2f » Design moment
Thickness of flange
Mu
t
Width of web bw -MM-
Effective depth of beam . . d
»X
0« i f'
of \1
\
Concrete strength
Yield strength of steel . . .
Span of beam
\
L
Use smallest value of "b"
© b = J2 sP an + b
© b -u
U = b' + 6t T ui

Required : © b = y center to center spacing of


a) Effective width of flange b
t) Depth of compression block . a beams
c) Steel area required As

You might also like